solve the equation .
X-8= 20 - 6(x + 7)

Answers

Answer 1
114 is the answer but if you want to know what X is then X is 12

Related Questions

can someone help me solve -2(8m+8)=-16

Answers

-2(8m+8)=-16

divide both sides of the equation by -2

8m+8 = 8

then cancel like terms on both sides

8m = 0

divide both sides by 8

m= 0

Which of the following statements is the most precise and correct to define a circle?

Answers

Answer:

THE SET OF ALL POINTS IN A PLANE THAT ARE THE SAME DISTANCE AWAY FROM SOME GIVEN POINT , WHICH WE CALL THE CENTER.

Step-by-step explanation:

Is 8 a solution to -2(x + 10) + 4 = -32? True or False

Answers

Step-by-step explanation:

-2(x + 10) + 4 = -32

-2(x + 10) = -32 -4

-2(x + 10) = - 36

x + 10 = (-36)/(-2)

x + 10 = 18

x = 18 - 10

x = 8

8 is a solution ... true

What is the range of the data? 2,8,7,8,6,15,6,3,8

Answers

Answer:

13

Step-by-step explanation:

The range of a data set is the difference between the greatest and least values in the set. In this case, the greatest value is 15 and the least value is 2 so the range is 15 - 2 = 13.

find three consecutive integers for which 3 times the sum of the first and the third is -342

Answers

Answer:

The integers are -58, -57, -56.

Step-by-step explanation:

The difference between two consecutive integers is 1.

If you let the smallest of the three consecutive integers equal x, then the next greater one is x + 1, and the greatest of the three is x + 2.

Integers: x, x + 1, x + 2

The sum of the first and third is: x + x + 2

3 times the sum of the first and third is: 3(x + x + 2)

That product equals -342.

3(x + x + 2) = -342

3(2x + 2) = -342

Divide both sides by 3.

2x + 2 = -114

Subtract 2 from both sides.

2x = -116

Divide both sides by 2.

x = -58

x + 1 = -57

x + 2 = -56

The integers are -58, -57, -56.

What is the value of 4s2+10 if s=9?

Answers

Answer:

Step-by-step explanation:

Is the 2 an exponent? if so:

Plug 9 into S

so rewrite as 4(9)^2+10=1306

If not: then 4(9)(2)+10= 82

9x+12x-21=27
How do I solve ?

Answers

You gotta combine like terms which would be 9x and 12x which gives you 21x .

The sum of the measures of the angles shown is 90°.
(5x)
(7x + 6)
10. Write an equation to determine the value of x.
11. Find the value of x.

Answers

Answer:

10). 5x + (7x + 6) = 90

11). x = 7

Step-by-step explanation:

From the picture attached,

Sum of given angles measures 90°

Measures of the angles are (5x)° and (7x + 6)°

Therefore, equation to determine the values of x will be,

5x + (7x + 6) = 90

By combining similar terms of the equation,

12x + 6 = 90

12x = 90 - 6

12x = 84

x = [tex]\frac{84}{12}[/tex]

x = 7

Therefore, by solving the equation value of x will be 7.

Brainliest for correct answer

Answers

Here. The paper gives a little bit of an explanation...

12. Find the area of the rectangle. + 2​

Answers

Answer:

The formula for the rectangle is base times the height. Since theres

no picture. SORRY!!

Step-by-step explanation:

what is the measure of X?
angles are not necessarily drawn to scale

Answers

Answer:

xº=71º

Step-by-step explanation:

Take the triangle BDH

Once the sum of interior angles of a triangle is 180º:

40º+31º+yº=180º

71º+yº=180º

yº=109º

Once yº and xº are supplementary angles (add up to 180º):

yº+xº=180º

109º+xº=180º

xº=71º

If ∠DBE is 40° and ∠BDC is 31° then the angle x will be equal to 71°.

What is triangle?

A triangle is a two dimensional figure having three sides, three angles and the sum of all the angles is equal to 180°.

How to find angle?

We have been given ∠DBE=40° and ∠BDC=31° and we have to find the angle ∠IHE which is x°.

Let the angle DHB equal to y.

In ΔBDH

∠DBH+∠BDH+Y=180 { Sum of all angles in a triangle is equal to 180}

40+31+y=180

71+y=180

y=180-71

y=109°

We know that sum of two angles on a line is equal to 180°.

So, x=180-109

x=71°

Hence the value of x is 71° if ∠BDC=31° and ∠DBE =40°.

Learn more about triangles at https://brainly.com/question/17335144

#SPJ2

You and a friend are heading to the movies. You each buy a ticket and you also want to buy some refreshments. The total cost of admission per person is, a, is $7.50 and you spent $15.50 on refreshments. Write an expression to represent the total cost of the trip. Then, find the total cost.

Answers

$7.50•2= $15.00
15.00+$15.50= $30.50 in total good luck :)))

Answer:

how did you find the answer of 1 ticket

Step-by-step explanation:

Which is the measurement of 144 ounces? A: 12 pounds B: 9 pounds C: 18 pounds D: 16 pounds

Answers

Answer:

B

Step-by-step explanation:

because the mass value is 16 x 9pounds equals 144

B is the correct answer bro and girl

Betty makes pies. to make 6 pies, she uses 7 1/2 cups of flour. How many cups of flour are needed to make one pie Please submit the improper fraction and a mixed number. If you don´t, be surprised when your answer gets reported <3

Answers

Answer:

1 1/4 or  5/4 cups of flour

Step-by-step explanation:

7 1/2 divided by 6 equals 1 1/4.

hope this helps you :)

Answer:yes

Step-by-step explanation:

9. If DF = 42, find DE.
7x+1 4x-3
DEF what is DE​

Answers

Answer:

29

Step-by-step explanation:

So DF = 42, And we know that DF = DE + EF

Therefore (7x+1) + (4x-3) = 42

11x -2 = 42

11x = 44

x = 4

Substitute x = 4 onto DE equation, so 7(4) +1

= 29.

The required measure of line segment DE is 29.

Given that,
A line segment DF is given,
E is a point on the  segment DF, where DE = 7x +1 and EF = 4x - 3
and DF = 42,
To determine the measure of DE.

What is a line?

A line is a straight curve showing the shortest distance between 2 points.

Here,
DF = DE + EF
From the given put the values of  DF = 42, DE = 7x +1 and EF = 4x - 3
42 = 7x + 1 + 4x - 3
42 = 11x - 2
11x = 44
x = 4

DE = 7x + 1
Put the value of x in above equation,
DE = 7 * 4  + 1
DE = 28 + 1
DE = 29

Thus, the required measure of the DE segment is 29.

Learn more about lines here:

brainly.com/question/2696693

#SPJ2

1.) Given N is between M and P. If MN = 3x +2, NP = 18, and MP = 5x find the length of each
segment

Answers

Step-by-step explanation:

MN=MP+PN

3X+2=5X-18

2X=20

X=10

MP=50

MN=32

NP=18

what is the value of x (x+2)(x-3)=0

Answers

Answer:

x = 3 or x = -2

Step-by-step explanation:

Solve for x over the real numbers:

(x + 2) (x - 3) = 0

Hint: | Find the roots of each term in the product separately.

Split into two equations:

x - 3 = 0 or x + 2 = 0

Hint: | Look at the first equation: Solve for x.

Add 3 to both sides:

x = 3 or x + 2 = 0

Hint: | Look at the second equation: Solve for x.

Subtract 2 from both sides:

Answer: x = 3 or x = -2

A food truck sells tacos for 6.50 each and burritos for 7.50 each. The food truck's revenue from selling a total of 164 tacos and burritos in one day is 1157.00. How many tacos and burritos were sold that day?

Answers

Answer:

12 burritos and 167 tacos

Step-by-step explanation: 164 x 6.5 = 1066

1157 - 1066 = 91

91/7.5 about 12

164 tacos and 12 burritos

The area of a parallelogram is 5 cm Squared. explain why you can find the area without knowing the dimensions of the parallelogram. Please help quick!

Answers

Answer:

because a square always has the same length on each side so yo just see what number multiplies to that.

Step-by-step explanation:

Please help me out! :(

Answers

Ans: option 2

By substituting each x and y value in respected equation and u will get the ans.

Hope this can help.

please help,, thankyou!

Answers

Answer:

Shown in the picture

Answer :

[tex]:\implies\bf\:m+n=18\:\longrightarrow\:(I)[/tex]

[tex]:\implies\sf\:2mn=-24\:\longrightarrow\:(II)[/tex]

[tex]:\implies\bf\:mn=-12\:\longrightarrow\:(III)[/tex]

Answer - A :

[tex]:\implies\tt\:\dfrac{5}{3m}+\dfrac{5}{3n}[/tex]

[tex]:\implies\tt\:\dfrac{15(m+n)}{9(mn)}[/tex]

[tex]:\implies\tt\:\dfrac{15(18)}{9(-12)}[/tex]

[tex]:\implies\tt\:-\dfrac{15}{6}[/tex]

[tex]:\implies\:\boxed{\bf{\purple{-\dfrac{5}{2}}}}[/tex]

Answer - B :

[tex]:\implies\tt\:\dfrac{4}{m^2}+\dfrac{4}{n^2}[/tex]

[tex]:\implies\tt\:\dfrac{4(m^2+n^2)}{(mn)^2}[/tex]

[tex]:\implies\tt\:\dfrac{4(m^2+2mn+n^2-2mn)}{(mn)^2}[/tex]

[tex]:\implies\tt\:\dfrac{4[(m+n)^2-2mn]}{(mn)^2}[/tex]

[tex]:\implies\tt\:\dfrac{4[(18)^2-(-24)]}{(-12)^2}[/tex]

[tex]:\implies\tt\:\dfrac{4(324+24)}{144}[/tex]

[tex]:\implies\tt\:\dfrac{348}{36}[/tex]

[tex]:\implies\:\boxed{\bf{\orange{\dfrac{29}{3}}}}[/tex]

Three fences on a ranch measure 15/16 mile, 7/8 mile, and 7/16 mile. Which is the best estimate of the total length of all three fences. *

Answers

Answer:

deez

nuts

Step-by-step explanation:

The answer is 345. Use these numbers 10, 3, 4, 5, 2, 1, 100 to get 345

Answers

Answer:

Step-by-step explanation:

100 + 100 + 100 + 10 + 10 + 10 + 10 + 5

The question is primarily asking you to use the terms given to get the number given. As there is no rule written for this one, it is easier. Usually, or later on, they may give you a sequence in which you will plug in terms to obtain the results. In cases like that, you must remember to follow PEMDAS.

PEMDAS =

Parenthesis

Exponents & Roots

Multiplication

Division

Addition

Subtraction

& is the order of operation.

~

Answer:  10 × 3 + 4 + 5 × 2 + 1 + 3 × 100

Step-by-step explanation:

10 × 3 + 4 + 5 × 2 + 1 + 3 × 100 = 345

Using PEMDAS, multiply first ... then add/subtract.

30 + 4 + 10 + 1 + 300 = 345

      34 + 10 + 1 + 300 = 345                  

              44 + 1 + 300 = 345

                   45 + 300 = 345      

                           345 = 345            

There are two diferent maps of Ohio.

◦The scale on the first map is 1 cm to 10 km. The distance from Cleveland to Cincinnati is 400km.
◦The scale on the second map is 1 cm to 50 km.

What is the distance from Cleveland to Cincinnati on the second map? Explain your reasoning.

Answers

The distance from Cleveland To Cincinnati on the second map is 8 cm because every cm is 50 km

(01.05 LC)Which fraction is equivalent to negative 4 over 6? A.) 6 /-4 B.) - 6/4 C.) 4 /-6 D.) 4/6 I NEED HALP OR ELSE IMANA DIE HERE PLSS HELPPPPP!!!!!!!!

Answers

Answer:

C

Step-by-step explanat:

Hope thats right

Answer: -4 over 9 hope this works

Step-by-step explanation:

For the function f(x) = −2(x + 3)2 − 1, identify the vertex, domain, and range.

Answers

Answer:

Vertex: (-3, -1)

Domain: All real numbers or (-∞, ∞)

Range: (-∞, -1]

Step-by-step explanation:

Quadratic Function: f(x) = a(bx - h)² + k

(h, k) is vertex

Step 1: Find vertex

f(x) = -2(x + 3)² - 1

h = -3

k = 1

Vertex = (-3, 1)

Step 2: Graph

When we graph, we should see our domain and range.

Domain is all x-values that can be inputted into the function.

Range is all y-values that can be outputted from the function.

Evaluate each expression. 6 ÷ 6 + z + x − y; use x = 2, y = 5, and z = 6

Answers

Answer:

4

Step-by-step explanation:

6/6+6+2-5=

In the diagram, what is the measure of WRS?
O 5°
O 7.5°
O 25°
O 37.5°

Answers

Answer : The measure of ∠WRS is 25°.

Step-by-step explanation :

As we know that vertically opposite angles are always equal.

In the given figure,

∠VRT = ∠WRS = (5x)°          (Vertically opposite angles)

And we know that the sum of all angle in a straight line is equal to 180°.

So,

∠TSR + ∠WRS = 180°

Given: ∠TSR = (25x + 30)°

∠TSR + ∠WRS = 180°

(25x + 30)° + (5x)° = 180°

30x + 30° = 180°

30x = 180° - 30°

30x = 150°

x = 5°

∠WRS = (5x)° = (5 × 5)°  = 25°  

Therefore, the measure of ∠WRS is 25°.

Answer: 25°

Step-by-step explanation:

Easy peasy go bedsy

how is the graph misleading?

Answers

Answer:

This graph is misleading because the colours of them are different.

Hope this helps!

Mikayla bought a bag of 84 pieces of candy and she wants to divide them up evenly and place into bags but doesn't won't more than 20 bags list the possible ways she can fill the bag

Answers

Answer:

[tex]1.0736 \times 10^{19}[/tex] ways

Step-by-step explanation:

Given that:

Mikayla bought a bag of 84 pieces of candy.

she wants to divide them evenly and place them into bag such that they are not more than 20 bags.

Therefore, the number of possible ways she can do that is ;

[tex]^{84}C__{20}[/tex]

= [tex]\dfrac{84!}{20!(84-20)!}[/tex]

= [tex]\dfrac{84!}{20!(64)!}[/tex]

[tex]= 1.0736 \times 10^{19}[/tex] ways

The possible ways she can fill the bag = [tex]1.0736 \times 10^{19}[/tex] ways

Other Questions
Which aspect of United States society did these men influence most in the late 1800s explain everything about an amoeba What is the origin of the energy that animals acquire by eating plants or other animals?A. Carbohydrates.B. Fats.C. Heat.D. Sugars.E. Sunlight. When is it appropriate to use the passive voice?when the person or thing performing the action is a proper nounwhen the person or thing performing the action comes before the verbIt is never appropriate to use the passive voice. when the person or thing performing the action is unimportant or unknown A cylinder has a volume of 120 cubic inches and a radiusof 6 inches. What is the height of the cylinder rounded tothe nearest inch? How much of a say did people in the American colonies have over their trade laws Look at the image below:Which of the following statements is true?Group of answer choicesThe figure is a molecule and a compound.The figure is a compound, but not a molecule.The figure is an atom and an element.The figure is a molecule and an element. twice the sum of a number and 400. Translate the verbal expression into an algebraic expression plz What is significant about September 15th? Why does Hispanic Heritage month start on the 15th instead of the 1st? The learning goals for this lesson are... Explain the mental, physical, social, and emotional needsnecessary to maintain good health. Define wellness. Describe various influences on a person'shealth and wellness. Find the value of c and YZ if Y is between X and Z.XY = 5.5. YZ = 2c. XZ = 8.9YZIN Tadashi's uncle passed away and left him a Realty Experts franchise. Tadashi is not a licensed agent or broker, nor does he know the first thing about the real estate business. He plans to sell his Realty Experts franchise to his friend Devonte, who recently got his real estate license. One of the advantages of owning a franchise is that you can decide to sell out to anyone you think is suitable for the business.a. Trueb. False what is how are u doing in spanish Given the vectors shown, find the sum (P+Q+R). Help!! Short and sweet question any help is appreciated! A fat contains which molecular components? A fat contains which molecular components? two fatty acids and one glycerol molecule three fatty acids and one steroid1 molecule three fatty acids only three fatty acids and one glycerol molecule What is your favorite class? Why? Which was not a crop grown by the English in their Colonies?wheatsoyricetobacco A group of 6 students was asked, " How many hours did you watch television last week?" Here are their responses 17 , 15 , 5 , 8 , 20 , 20 Find the mean number of hours for these students. If necessary, round your answer to the nearest tenth. The data below were obtained from an experiment were participants were given drinks with or without caffeine and then asked to tap their fingers. The data for 20 participants are below. Assume the number of taps per minute is normally distributed. The variance is unknown. Find a 95% CI for number of taps. Identify the pivot function used. 246 242 248 245 250 244 252 248 248 247 250 248 246 242 248 244 245 246 250 242